PT30.S4.Q20 - consumer advocate: the introduction of a new drug

dcdcdcdcdcdcdcdcdcdc Alum Member
edited March 2016 in Logical Reasoning 382 karma
https://7sage.com/lesson/introducing-new-drugs-strengthen-question

Looking to get a better idea of the logic on answer choice E, which is an incorrect answer choice.
I understand that A is the correct choice because reasoning from an example which is already the best case compared to alternatives strengthens the argument, given the premises.

Comment from @DumbHollywoodActor was helpful in shedding some light on E's logic:

(E) mixes up the logic. If you take the contrapositive, you can see it more clearly: “if most new drugs shouldn’t be on the market, then the new antihistamine shouldn’t be on the market.” The argument provides the necessary condition, but that doesn’t mean it gets to conclude the sufficient condition.
However, I would like to understand this statement better: The argument provides the necessary condition, but that doesn’t mean it gets to conclude the sufficient condition.

We must accept the premises and the conclusion as true for LSAT questions. I get that if we accept the premise as true, that is affirming the necessary condition of the logic in choice E. But, if we accept the conclusion as true, that also satisfies the sufficient condition of the logic.

Put simply, after reading the argument I am left with these two true pieces of information:
P: antihistamine should not be on the market (A)
C: these new drugs should not be on the market (B)
then E gives me this logic of "these new drugs should not be on the market" --> "antihistamine should not be on the market"

So, I'm left looking at that B--> A statement, and holding A and B in my hands, with no understanding of where to plug them in. If I plug B in, then I get A, but apparently that is not the correct answer.

Is it because the reasoning in this argument is inductive (that is, moving from a specific example to a general rule) and so it isn't helpful to say that "this general rule is the case" therefore "this specific examples is the case," since the argument is not applying a rule, but rather trying to support one?

Comments

  • runiggyrunruniggyrun Alum Inactive Sage Inactive ⭐
    edited March 2016 2481 karma
    You are right that for answer E if you plug B in (new drugs shouldn't be on the market) you get A (histamine shouldn't be on the market), but the stimulus doesn't give you B. It gives you A. The premise is that the author thinks the new histamine shouldn't be on the market. And plugging in A doesn't give you B - that would be an illegal reversal of logic.
    In other words, in your B-->A argument, you are given the necessary as a premise. Fulfilling the necessary doesn't mean you get the sufficient. It only works the other way around - fulfilling the sufficient triggers the necessary.
    I think that's @DumbHollywoodActor's point as well.
  • DumbHollywoodActorDumbHollywoodActor Alum Inactive ⭐
    7468 karma
    I think the next statement I said in that comment is telling:
    "When you get through the logic games portion of the curriculum, you’ll see this super clearly that affirming the necessary condition renders the conditional statement irrelevant.”

    Even though it’s a little past where you are, watch this video: https://7sage.com/lesson/conditional-rules-trigger-v-irrelevant

    I think once you’ve really grappled with it, you’ll understand what I and the always brilliant @runiggyrun mean.
  • dcdcdcdcdcdcdcdcdcdc Alum Member
    382 karma
    Thanks! I'll work on that lesson now and see what comes of it.
  • dcdcdcdcdcdcdcdcdcdc Alum Member
    382 karma
    @runiggyrun said:
    the stimulus doesn't give you B. It gives you A.
    Isn't B the conclusion of the stimulus?
    So, the stimulus is giving us both A and B, but the difference is A is supplied as a premise?


    -----
    Either way, I've now diagrammed the stimulus/argument as
    A (antihistamine should not be on the market) --> B (new drugs should not be on the market)

    whereas I diagrammed E as
    B (new drugs should not be on the market) --> A (antihistamine should not be on the market)

    The affirming the necessary error is clear in this form.
  • runiggyrunruniggyrun Alum Inactive Sage Inactive ⭐
    2481 karma
    Yes, by "the stimulus is giving you A" I mean A is given as a premise. Sorry about being unclear.
  • dcdcdcdcdcdcdcdcdcdc Alum Member
    edited March 2016 382 karma
    Ahh ok, at 7:10 into the video explanation of that strengthen question JY mirrors the lesson on the in-out games and it clicked.

    E is giving us A, but the argument is giving us not A, so rule irrelevant, hence E is wrong.

    Is it problematic at all that the sufficient condition in answer choice E is seems to contradict the premise of the argument? Taking both to be true kind of hurts my head, but since this a strengthen question, don't really see an issue.

    Thanks again!
Sign In or Register to comment.